USA TSTST 2015, 1

Proprietà dei numeri razionali, reali e complessi. Studio di polinomi, successioni, disuguaglianze e funzioni.
Rispondi
Veritasium
Messaggi: 206
Iscritto il: 30/03/2015, 20:36

USA TSTST 2015, 1

Messaggio da Veritasium »

Sia [tex]a_1, ..., a_n[/tex] un sequenza di reali e sia [tex]m < n[/tex] un intero positivo. Diciamo che un indice [tex]k[/tex] con [tex]1 \le k \le n[/tex] è elettronegativo se esiste [tex]l, 1\le l \le m[/tex] tale che [tex]a_k + a_{k + 1} + ... + a_{k + l - 1} \ge 0[/tex] con gli indici ovviamente modulo [tex]n[/tex]. Dimostrare che, se [tex]E[/tex] è l'insieme degli indici elettronegativi, [tex]\sum_{k \in E} a_k \ge 0[/tex]

Edit: editato
Ultima modifica di Veritasium il 01/06/2016, 20:09, modificato 1 volta in totale.
Avatar utente
Giovanni98
Messaggi: 1255
Iscritto il: 27/11/2014, 14:30

Re: USA TSTST 2015, 1

Messaggio da Giovanni98 »

Soluzione :
Testo nascosto:
Definisco $l_k := \min$ intero $0\leq l<n$ tale che $\sum_{i=0}^l a_{k+i} \ge 0$. Notiamo che se $k$ è tale che $a_k \ge 0$ allora $k$ è elettronegativo, quindi sostanzialmente dobbiamo dimostrare che la somma degli $a_i$ non negativi è $\ge$ della somma degli $a_i$ negativi tali che $i$ è elettronegativo.

Si prenda il primo $a_i$ negativo con indice $i$ elettronegativo (cioè $i = \min$ intero $\ge 1$ e $\le n$ tale che $i$ sia elettronegativo) e sostituiamo $ a_i,a_{i+1},\cdots,a_{i+l_i}$ con $S_t := a_i+a_{i+1}+\cdots+a_{i+l_i}$ (dove $t$ sta ad indicare il numero di volte che abbiamo fatto questo ragionamento) che sappiamo essere $\ge 0$ e dimostriamo che se esiste un indice $j$ tale che $i < j < i+l_i$ allora $a_j + a_{j+1} + \cdots + a_{i+l_i} \ge 0$. Supponiamo per assurdo che non sia vero (1), noi sappiamo che $a_i+a_{i+1}+\cdots+a_{i+l_i} = a_i + a_{i+1} + \cdots + a_j + a_{j+1} + \cdots + a_{i+l_i}$ è $\ge 0$, ma se ora vale (1) allora chiaramente $l_i$ non è il minimo intero tale che $\sum_{j=0}^{l_i} a_{i+j} \ge 0$, il che va contro le ipotesi.

Se $\sum_{k \in E} a_k < 0$ allora l'ultimo elemento della successione $S_i$ che indichiamo con $s$ deve essere negativo, in quanto ovviamente $$s \le \sum_{k \in E} a_k $$ma questo è ovviamente assurdo per come definiamo i membri della successione $S_i$, quindi la tesi è dimostrata.
Testo nascosto:
Il testo non è riportato correttamente, https://www.artofproblemsolving.com/com ... 2015_tstst
Veritasium
Messaggi: 206
Iscritto il: 30/03/2015, 20:36

Re: USA TSTST 2015, 1

Messaggio da Veritasium »

Buona, uguale alla mia.
Chissà se prima o poi riuscirò a scrivere correttamente un testo :roll:
Rispondi